Please answer correctly !!!!!!!! Will mark brainliest !!!!!!!!!!!

Please Answer Correctly !!!!!!!! Will Mark Brainliest !!!!!!!!!!!

Answers

Answer 1

Answer:-5-1 c

Step-by-step explanation:


Related Questions

Katherine earned 84, 92, 84, 75, and 70 on her first 5 tests.
What is the minimum grade Katherine needs to earn on the next test to have a mean of 84?

Answers

Answer:

D

Step-by-step explanation:

Okay, so the best way to find is too plug in the answer choices.

to find the mean you: add all the numbers up and then, you divide it by however many numbers there are.

minimum meaning exactly 84 no greater!

Start with letter choice A

84+92+84+75+70+81=486

486/6=81 which is  below minimum

Next is letter choice B

84+92+84+75+70+84=489

84/6=81.5 which is below minimum

Next is letter choice C

84+92+84+75+70+95=500

500/6=83.3 which is below minimum

Next is letter choice D

84+92+84+75+70+99=504

504/6=84 which is the correct answer

pls mark me brainliest

25x⁴+4x⁴y²+4y⁴ please answer this question ​

Answers

Answer:

(5x²+2y² + 2xy)(5x²+2y² - 2xy)

Step-by-step explanation:

It must be: 25x⁴+4x²y²+4y⁴

Then solution is:

25x⁴+4x²y²+4y⁴= (5x²)²+2*5x²*2y²+(2y²)² - 16x²y²= (5x²+2y²)² - (4xy)²= (5x²+2y² + 2xy)(5x²+2y² - 2xy)

Whats the square root of 10000

Answers

Answer: 100

Step-by-step explanation:

Answer:

The square root of 10000 is 100

Step-by-step explanation:

If x=4 and y=5 and z=-2 what is the answer for this equation: 3x + 1

Answers

Answer:

13

Step-by-step explanation:

We only need to pay attention to the x. We multiply 3 and 4 to get 12. Then we add 1 to get 13.

Answer:

13

Step-by-step explanation:

3x + 1

Let x = 4

3*4 +1

Multiply

12+1

13

Use the table of values to find the function's values. If x = 0, then f(0) = . If f(x) = 27, then x =

Answers

If x = 0, then the value of the function f(0) will be negative 15. And if the value of the function f(x) is 27. Then the value of x will be 3.

What is a function?

A function is a statement, rule, or law that establishes the connection between two variables. In mathematics, functions are everywhere and are necessary for constructing physical connections.

The table is given below.

If x = 0, then the value of the function f(0) will be

f(0) = -15

And if the value of the function f(x) is 27. Then the value of x will be

x = 3

More about the function link is given below.

https://brainly.com/question/5245372

#SPJ1

Answer:

if x = 0, then f(0) = -15 and if f(x) = 27, then x = 3

Step-by-step explanation:

i look at the table for 0 and 27 and then look at the numbers next to it

Griffin ordered a pair of sneakers online. He had a $22 credit that he applied toward the purchase, and then he used a credit card to pay for the rest of the cost. If the shoes cost $65, how much did Griffin charge to his credit card when he bought the sneakers?

Answers

Answer:$43

Step-by-step explanation:

65 -22 =43

joy and Peter were each walking along a straight road. Joy walked 1800 meters in 1/3 hour. Peter walked 1/2 of joy's distance in 1/5 hour.
1. What was joy's average walking speed
2. How much did Peter walk in 1/5 hour
3. What is peter's average walking speed
4. Joy walks how much meters/hours faster, slower, or as fast as

Answers

The answer is 3 because it says how much joy walked and not Peter , you’re trying to figure out what’s peters average walking speed

A sector with a radius of 15cm has a central angle measure of 8pi/5 (in radians)

Answers

Answer:

565.49 cm2

Step-by-step explanation:

The area of a circle is given by:

Area = pi * r^2

And the whole area represents a central angle of 2pi.

Then, to find the area of this sector, we just need to use a rule of three:

area of pi * r^2 -> central angle of 2pi

area of x -> central angle of 8pi/5

x * 2pi = pi * r^2 * 8pi/5

2x = r^2 * 8pi/5

x = r^2 * 4pi/5 = 15^2 * 4pi/5 = 565.49 cm2

Answer:180pi cm^2

Step-by-step explanation:

Cuz I got it right

20 POINTS !! PLEASE ANSWER THIS QUESTION !! WILL GIVE BRAINLIEST !!

Answers

Answer:

c. 81

Step-by-step explanation:

(x^3)^1/6= √81

x^(3*1/6)= √81

x^(1/2)= √81

√x=√81

x=81

The answer is D.6561

5y + 3 > −7y + 13 Multi-step linear inequalities

Answers

Answer:

y > [tex]\frac{5}{6}[/tex]

Step-by-step explanation:

Given

5y + 3 > - 7y + 13 ( add 7y to both sides )

12y + 3 > 13 ( subtract 3 from both sides )

12y > 10 ( divide both sides by 12 )

y > [tex]\frac{10}{12}[/tex] , that is

y > [tex]\frac{5}{6}[/tex]

Please answer correctly !!!!!!!!! Will mark brainliest !!!!!!!!!!!!

Answers

Answer: 33

Step-by-step explanation:

there ages m and b.

m = b + 12

m - 17 = 4(b - 17) m = 4b - 51

3b = 63

b = 21

m = 21 + 12 = 33

Hope this helps :)

Answer: Michael is 33 years old.

Step-by-step explantation: Okay, think of it like this, Michael's age for right now is x since we still do not know Michael's age. Pay close attention to the first sentence. What does it say?  m = b + 2. Now pay attention to the second sentence. What does it say? m - 17 = 4 ⋅ (b = 17). Now, let's move on to the system of equations. As you can see here, the system of equations is

{m = b + 12

{m - 17  = 4 ⋅ (b - 17}.

Now for more equations.

m - 17 = 4b - 68

m = 4b - 68 + 17

m = 4b - 51

m - 4b = -51

-

b + 12 - 4b = -51, through the substitution from the first equation we saw

-3b = -51 - 12

3b = 51 + 12

b = 51/3 + 12/3

b = 17 + 4

b = 21

-

m = 33.

what is the volume of this container ​

Answers

Answer:

1468 in³

Step-by-step explanation:

First get get the total volume of the rectangular prism:

length l = 10 in

width w = 15 in

height h = 10 in

diagonal d = 20.6155281 in

total surface area Stot = 800 in²

lateral surface area Slat = 500 in²

top surface area Stop = 150 in²

bottom surface area Sbot = 150 in²

volume V = 1500 in

Now get get the volume from the missing rectangular prism:

ength l = 4 in

width w = 4 in

height h = 2 in

diagonal d = 6 in

total surface area Stot = 64 in²

lateral surface area Slat = 32 in²

top surface area Stop = 16 in²

bottom surface area Sbot = 16 in³

volume V = 32 in³

So 1500in³- 32 in³ =1468 in³

Answer:

1468

Step-by-step explanation:

10*10*15 = 1500

1500 - (4*4*2) = 1468

1500-32=1468

Therefore the answer is 1468

Put the following equation in slope-intercept form: x+2y=16​

Answers

Answer:

y= -x/2+8

Step-by-step explanation:

Cause I know UwU

Find the area of the semicircle.

Answers

The answer is 39.27

To divide 1.89 ÷ 0.9, we need to change it. Choose the correct problem below to show what the “new” look would be.
A. 189 ÷ 9
B. 18.9 ÷ 0.9
C. 1.89 ÷ 9
D. 18.9 ÷ 9

Answers

Answer:

D. 18.9 ÷ 9

Step-by-step explanation:

we need to divide

1.89 ÷ 0.9 but write it in different form

so ,we need to eliminate decimal from 0.9

as 0.9*10 = 9

thus,we multiply both  1.89 and  0.9 with 10, then we will have

(1.89*10) ÷ (0.9*10)

=> 18.9 ÷ 9

Thus, based on above calculation new look would be D. 18.9 ÷ 9

All natural numbers excluding 1 and prime numbers are best described as?
___________________

Answers

Answer:

Composite Numbers.

Y
U(7,6)
T(3,4)
X
0
(k, k+8)
The figure shows a straight line passing through the points T(3, 4) and U(7,6).
(a) Find the gradient of TU.
(b) Find the equation of the line TU.
(c) A point (k, k+ 8) lies on the line TU produced. Find the value of k.​

Answers

Answer:

a) 1/2

b) y = 0.5x + 2,5

c) k = -3

Step-by-step explanation:

Given are points T(3, 4) and U(7,6).

To find the gradient of TU, you look at the incline in value in the y-direction, divided by the incline in value in the x-diretion.

(Uy- Ty) / (Ux- Tx)

substitute the values given and you get:

(6-4) / (7-3)

2 / 4

This gives you the answer on question a) The gradient of TU = 1/2

Any line is of the form y = ax + b with a = the gradient found in answer a) an b is the the value on the y-coordinate on the y axis, where the line TU intersects with the y-axis.

The whole next part is my attempt to explain to you something about getting the answer for b. Sorry if it is so long, but please take the time to read the explanation, and I hope it will help you to understand what is going on.

The gradient of TU = 1/2 which means that if you start on any point on the line, and then you go 2 units to the right and 1 up you will end upon another point on the line. That is because the quotient is the same. 2/4 = 1/2 = 0,5/1

The last quotient, 0,5/1 , also has the resulting value of 0,5.

This means that if you start on any point on the line, and then you go 1 unit to the right and 0.5 up, you will be on another point on the line.

ATTENTION : This means also, that if you start on any point on the line TU, and then you go 1 unit to the LEFT and 0.5 DOWN, you will be on another point on the line TU. If you understand this, than you have learned something important!

We need to know where the line TU intersects with the y-axis, and now we can use this to find that value?

Since point T(3,4) lies 3 units to the right of the y-axis, we need to go 3 units to the LEFT. But remember the incline of 0.5/1 ... For every unit you go to the left you need to go 0,5 unit DOWN to remain on the line.

If you go 3vunits to the left, how many units do you need to go down?

3 * 0.5 units which is 1.5.

Point T has an y coordinate of 4 so ... what will the y coordinate be if you go 3 units to the left?

4 - (1.5) = 2.5

So now finally we can write down the answer to question b).

y = 0.5x + 2,5

c) (k, k+ 8) lies on the line TU. So substitute that in y = 0.5x + 2,5

inplace of x you write kinplace of y you write k+8

y = 0.5 * ( x ) + 2,5

k+8 = 0.5 * ( k + 8 ) + 2,5

k+8 = 0.5*k + 4 + 2,5

Bring all k values to the left of the equality sign, and all number values to the right.

k - 0.5*k = 4 + 2,5 - 8

0.5*k = 6,5 - 8

0.5*k = -1,5

multiply left and right of the = sign by 2 and you get your answer on question c).

k = -3

Part A


The dot plot shows the daily lunch money of a group of students.


Each dot represents one student.


1


6


2 3 4 5


Daily Pocket Money ($)


What is the mean daily pocket money?

Answers

*diagram of the dot plot is attached below

Answer:

$2.15

Step-by-step Explanation:

Mean daily pocket money or average daily pocket if the group if students shown in the for plot that is attached below = the sum of the pocket money received by the students ÷ the total number of students.

From the diagram of the dot box attached below, the total number if students = 13 (each dot represents a student)

Sum of the pocket money received by the students = 4(1) + 5(2) + 3(2) + 4(2)

= 4 + 10 + 6 + 8

Sum = 28

Therefore, mean daily pocket money = 28 ÷ 13 = $2.15

Which line is a slop that is positive

Answers

Answer: A

Step-by-step explanation:

Answer:

the correct answer i believe is A. :)

Which of the matrixes can be added together? Choose all that apply.

Answers

Answer: A and C

Both matrices are 1 x 4 matrices. This notation says there is 1 row and 4 columns. The number of rows must match up, as well as the number of columns, in order for matrix addition to be possible. This is so the corresponding elements pair up and add together. For instance, the 5 and -2 pair up and add together for matrices A and C.

A group of 6th-grade students ran the 50-yard dash. Their times are
shown in the line plot below. What was the median time for the 50-yard
dash?
Time on 50-yard Dash
(in seconds)

Answers

Answer:

the median time for the 50 yard dash is 8 seconds

Step-by-step explanation:

hope this helps :)

Really stuck on this please help!

Answers

Answer:

4) BC=30.14

11)XZY≈47.5°

Step-by-step explanation:

4) we use cosine law because we have two length known and one angle known

BC²=AB²+AC²-2ABcos55°

BC²=23²+20²-2*23*20cos55°=908.6

BC=√908.6= 30.14 cm

11) we use cosinee law because three length known

cosXZY=(22²+20²-17²)/(2×22×20)

cosXZY=0.676

XZY^= cos^-1 0.676

XZY^=47.5°

Which Earth feature was created in a similar way to how much of the solar system was formed? A. Meteor Crater B. the Grand Canyon C. the Petrified National Forest D. Half Dome in Yosemite National Park

Answers

Answer: i think its b.

Step-by-step explanation:

Lines m and n are parallel. What is Measure of angle 2?

Answers

I’m pretty sure it is 106 because 180 minus 74 which

Answer:

67

Step-by-step explanation:

PLEEEEEEEEEASE HEEEEEEEEELP

In a large harbour there are many docks, labeled alphabetically, which are built side by side in a straight line. Dock A and Dock K are located 2.3 km apart along this line. There is a Dock L on an island outside the harbour such that is 45° and is 64°. How far is the island dock away from each of the other two docks?

Answers

Answer:

distance island dock to Dock A = 4.99 km

distance island dock to Dock K = 6.35 km

Step-by-step explanation:

Always make a scetch to visualize the situation.

You need to construct two triangle both with a streight angle, so you can use Pythagoras to calculate the unknown distances between the island dock L, and each of the other two docks A an K.

I chose to introduce an extra letter, the letter C. In total you have the letters A K L and the letter C.

The letter C has a streight angle of 90° between ACL and it has the same streight angle of 90° with KCL. It is crucial that you see that the distance of LC is exactly the same in triangle LAC and that LC has exactly the same distance in the other triangleLKC.

The distance between AK = 2.3 km.

I define the distance between K and point C as 2.3 + x, because the distance x is unknown.

KC = 2.3 + x

Further more, when you make a picture, you can see that the distance between A and point C = x.

From such a picture, it would show clearly, that K is further away in respect to L then point A. From the picture it would be clear that the angle of LKC is smaller then the angle of LAC, so LKC = 45° and LAC = 64°.

Because angle LKC = 45° and we choose C to have an angle of 90°, the TRIANGLE LKC must be a special triangle... In any triangle, the sum of the three angles together, must add up to 180° .

If that is true, then we have 45 + 90 + 45 (because that adds up to 180). Now that means triangle LKC must have two equal sides (because of the same angels of 45° ).

So we know the distance KC = LC and we already defined KC = 2.3 + x.

Now we know enough to solve the problem.

AK = 2.3 km

angle of LKC = 45°

angle of LAC = 64°

AC = x

KC = 2.3 + x

LC = KC

LC = 2.3 + x

Try to calculate the distance x by using tan. After that you can use Pythagoras to find the other distances.

tan(LKC) = ( LC ) / ( KC )

tan(LKC) = ( x+2.3 ) / ( x+2.3 )

That is not helpful. Let's try the other triangle...

tan(LAC) = LC / AC

tan(LAC) = ( x+2.3 ) / x

tan(64) = ( x+2.3 ) / x

Solve the equation which means you try to find the value for x.

x * tan(64) = ( x+2.3 )

tan(64) * x -x = 2.3

tan(64) * x - 1* x = 2.3

Try to get x outside of the braquets...

x* ( tan(64) - 1 ) = 2.3

x* (2.0503038415793 - 1 ) = 2.3

1.0503038415793 * x = 2.3

x = 2.3 / 1.0503038415793

x = 2.19

Now use Pythagoras a² + b² = c² in triangle LAC to find distance LA.

LA² = AC² + LC²

AC = x = 2.19

LC = 2.3 + x = 4.39

LA² = 2.19² + 4.39²

LA = SQRT( 4.79 + 20.16 )

LA = SQRT( 24.95 )

LA = 4.99 km

Now use Pythagoras a² + b² = c² in triangle LKC to find distance LK.

LK² = KC² + LC²

KC = 2.3 + x = 4.39

LC = 2.3 + x = 4.39

LK² = 4.39² + 4.39²

LK = SQRT( 20.16 + 20.16 )

LK = SQRT( 40.32 )

LK = 6.35 km

What is the ratio of the volume of Cylinder A to the volume of Cylinder B? If cylinder A has a radius of 3 and a height of 4 and cylinder B has a radius of 4 and a height of 3

Answers

Answer:

3:4

Step-by-step explanation:

Volume of cylinder A = V= π[tex]r^{2}[/tex]h = r,3 h,4 = [tex]\pi[/tex]36

Volume of cylinder B = V= π[tex]r^{2}[/tex]h = r,4 h,3 = [tex]\pi[/tex]48

Ratio of the volume = [tex]\frac{36}{48}[/tex]

= [tex]\frac{18}{24}[/tex]

= [tex]\frac{9}{12}[/tex]

= [tex]\frac{3}{4}[/tex]

= 3:4

Hope it helps!

Mark as brainliest!

A line passed through the origin and has a slope of 1/2.

Answers

Answer:

D. (2, 1)

Step-by-step explanation:

slope = 1/2

y-intercept = 0

Equation: y = 1/2 x

Choice A:

x = 0

y = (1/2)(0) = 0

Choice A does not work.

Choice B:

x = 1/2

y = (1/2)(1/2) = 1/4

Choice B does not work.

Choice C:

x = 1

y = (1/2)(1) = 1/2

Choice C does not work.

Choice D:

x = 2

y = (1/2)(1) = 1

Choice D works.

Answer: D. (2, 1)


Plsss he helppp fasttttt

Answers

Answer:

c d

Step-by-step explanation:

no explanation

take out a minus from first two fractions

D
Explenation: they are negative in the first one and just listen to me look distribute the negative at the front and makes the fractions negative

Can someone help me with this

Answers

Answer:

x=5

Step-by-step explanation:

2x+10=20

   -10   -10

2x=10

then divide both sides by 2

x=5

Answer:

x = 5

Step-by-step explanation:

Given

2x + 10 = 20

Subtract 10 to both sides

(2x + 10) -10 = 20 - 10

2x = 10

Divide both sides by 2

[tex]\frac{2x}{2} =\frac{10}{2}[/tex]

x=5

Check your work. Substituted 5 for x

2x + 10 = 20

2(5) + 10 =20

10 + 10 = 20

20 = 20

What is the standard notation of 415 km?

Answers

Answer:

The answer is 415 because it is already in standard notation.

Other Questions
Which inequality is represented by this graph? Tu nas pas de cousins? Senator Roy Blunt of Missouri says, "Im philosophically opposed to the federal government taking over elections. It is a bad idea. Im pretty flexible about the amount of money, but Im not flexible about a federal takeover of the election process itself." Do you agree? Should the federal government require all states to provide vote-by-mail options or should it be a state-by-state decision? Using what you have learned in the lesson and gathered from your survey, write a conclusion paragraph analyzing your results in your conclusion. Be sure to address the following questions:- Identify the trait that you believe to be dominant for each of the features surveyed. Explain your reasoning. - It is possible that a recessive trait in a survey such as this one may have a greater total number than its dominant counterpart. Explain how that might happen in an investigation like this.- How might surveying a different number of people or different population of people possibly change the percentages you calculated? My data tables are in the picture. Find the center, radius, and intercepts of the circle given below and then sketch the graph of the circle.x+(y-1)^2=1 Which is best supported by the data in the chart? A: Current W flows at a higher rate than Current Z. B: Current Y flows at a lower rate than Current X. C: Current X has a lower potential difference than Current Y. D: Current Z has a greater potential difference than Current W. Telephone pole breaks and falls as shown. To the nearest foot, what was theoriginal height of the pole? they are 7 ft and 10 ft pls explain how u got the answer and the choices r a.19ft. b.17ft. c.20ft. d.18 ft pls find the answer for me but its a triangle and in the bottom it has number 10 and in the long side of the triangle is 7 Whats the correct answer for this? Combine the following sentences using appropriate conjunctions to have just one sentence. 2.a) One effect of studying abroad is a students greater understanding. b) The effect is important. c) The understanding is of an educational system. d) The system is different Rectangle ABCD is graphed in the coordinate plane. The following are the vertices of the rectangle: A(3,2), B(6, -2), C(6, 5) D(3,5). What is the area of this rectangle What is one thing you could do today to help you and our friends delete digital drama? Which journal entry reflects the adjusting entry needed on December 31?:In November, BOC received a $5,000 cash deposit from a customer for custom-build goods that will be delivered in January (BOC recorded an entry for this $5,000 in November). Now, it is December 31, the end of the fiscal year.Dr. Unearned Revenue 5,000Cr. Inventory 5,000No entry needed.Dr. Cash 5,000Cr. Revenue 5,000Dr. Advances from Customers 5,000Cr. Revenue 5,000Dr. Unearned Revenue 5,000Cr. Revenue 5,000 do what you gotta do. helppp meh What is the perimeter of triangle PQR? How did the Middle Ages fall due to the feudal system? An open window is at the height of 15 ft above the ground. A ladder is placed 8 ft away from the side of the house. What will be the length of ladder needed to reach to the window of the house? Describe fully the single transformation that maps triangle a onto triangle b A rectangular swimming pool is 17 meters long, 13 1/2 meters wide, and 2 1/2 meters deep. What is its volume? need help!! plzz help asap A process is in control with mean 50 and standard deviation of 2. The upper specification limit for the product being produced is 60 and the lower specification limit is 42. What is the value of Cpk?